Show that in any field holds $((x^r-1),(x^s-1)) = x^d-1 iff d= gcd(r,s)$












0












$begingroup$



Show that in any field holds $((x^r-1),(x^s-1)) = x^d-1 iff d= gcd(r,s)$.




I define $r=ad$ and $s=bd$, then we have $((x^{ad}-1),(x^{bd}-1)) = x^d-1$, but the annoying thing for me to proceed are those "$-1$".



From literature:




For non-negative integers a and b, where a and b are not both zero,
provable by considering the Euclidean algorithm in base n we know
$gcd(n^a − 1, n^b − 1) = n^{gcd(a,b)} − 1$.




Some hints for a proof that doesn't use the Euclidean Algorithm?










share|cite|improve this question











$endgroup$








  • 1




    $begingroup$
    math.stackexchange.com/questions/7473/…
    $endgroup$
    – lab bhattacharjee
    Dec 18 '18 at 9:39










  • $begingroup$
    I can't understand this passage; if $n > m$, then $$gcd(a^n - 1, a^m - 1) =color{red}{ gcd(a^n - 1, a^n - a^{n-m}) = gcd(a^{n-m} - 1, a^m - 1)}$$
    $endgroup$
    – Alessar
    Dec 18 '18 at 10:02






  • 1




    $begingroup$
    If $d$ divide both, $d$ must divide $a^n-a^m=a^m(a^{n-m}-1)$ But $dnmid a^m$
    $endgroup$
    – lab bhattacharjee
    Dec 18 '18 at 10:08










  • $begingroup$
    In my case, your $d$ is my $x^d-1$, right? Sorry I want to understand clearly
    $endgroup$
    – Alessar
    Dec 18 '18 at 10:23








  • 1




    $begingroup$
    I've stated common divisor, ultimately this leads to the greatest common divisor
    $endgroup$
    – lab bhattacharjee
    Dec 18 '18 at 10:25
















0












$begingroup$



Show that in any field holds $((x^r-1),(x^s-1)) = x^d-1 iff d= gcd(r,s)$.




I define $r=ad$ and $s=bd$, then we have $((x^{ad}-1),(x^{bd}-1)) = x^d-1$, but the annoying thing for me to proceed are those "$-1$".



From literature:




For non-negative integers a and b, where a and b are not both zero,
provable by considering the Euclidean algorithm in base n we know
$gcd(n^a − 1, n^b − 1) = n^{gcd(a,b)} − 1$.




Some hints for a proof that doesn't use the Euclidean Algorithm?










share|cite|improve this question











$endgroup$








  • 1




    $begingroup$
    math.stackexchange.com/questions/7473/…
    $endgroup$
    – lab bhattacharjee
    Dec 18 '18 at 9:39










  • $begingroup$
    I can't understand this passage; if $n > m$, then $$gcd(a^n - 1, a^m - 1) =color{red}{ gcd(a^n - 1, a^n - a^{n-m}) = gcd(a^{n-m} - 1, a^m - 1)}$$
    $endgroup$
    – Alessar
    Dec 18 '18 at 10:02






  • 1




    $begingroup$
    If $d$ divide both, $d$ must divide $a^n-a^m=a^m(a^{n-m}-1)$ But $dnmid a^m$
    $endgroup$
    – lab bhattacharjee
    Dec 18 '18 at 10:08










  • $begingroup$
    In my case, your $d$ is my $x^d-1$, right? Sorry I want to understand clearly
    $endgroup$
    – Alessar
    Dec 18 '18 at 10:23








  • 1




    $begingroup$
    I've stated common divisor, ultimately this leads to the greatest common divisor
    $endgroup$
    – lab bhattacharjee
    Dec 18 '18 at 10:25














0












0








0





$begingroup$



Show that in any field holds $((x^r-1),(x^s-1)) = x^d-1 iff d= gcd(r,s)$.




I define $r=ad$ and $s=bd$, then we have $((x^{ad}-1),(x^{bd}-1)) = x^d-1$, but the annoying thing for me to proceed are those "$-1$".



From literature:




For non-negative integers a and b, where a and b are not both zero,
provable by considering the Euclidean algorithm in base n we know
$gcd(n^a − 1, n^b − 1) = n^{gcd(a,b)} − 1$.




Some hints for a proof that doesn't use the Euclidean Algorithm?










share|cite|improve this question











$endgroup$





Show that in any field holds $((x^r-1),(x^s-1)) = x^d-1 iff d= gcd(r,s)$.




I define $r=ad$ and $s=bd$, then we have $((x^{ad}-1),(x^{bd}-1)) = x^d-1$, but the annoying thing for me to proceed are those "$-1$".



From literature:




For non-negative integers a and b, where a and b are not both zero,
provable by considering the Euclidean algorithm in base n we know
$gcd(n^a − 1, n^b − 1) = n^{gcd(a,b)} − 1$.




Some hints for a proof that doesn't use the Euclidean Algorithm?







abstract-algebra field-theory






share|cite|improve this question















share|cite|improve this question













share|cite|improve this question




share|cite|improve this question








edited Dec 21 '18 at 17:42









Shaun

9,789113684




9,789113684










asked Dec 18 '18 at 9:33









AlessarAlessar

313115




313115








  • 1




    $begingroup$
    math.stackexchange.com/questions/7473/…
    $endgroup$
    – lab bhattacharjee
    Dec 18 '18 at 9:39










  • $begingroup$
    I can't understand this passage; if $n > m$, then $$gcd(a^n - 1, a^m - 1) =color{red}{ gcd(a^n - 1, a^n - a^{n-m}) = gcd(a^{n-m} - 1, a^m - 1)}$$
    $endgroup$
    – Alessar
    Dec 18 '18 at 10:02






  • 1




    $begingroup$
    If $d$ divide both, $d$ must divide $a^n-a^m=a^m(a^{n-m}-1)$ But $dnmid a^m$
    $endgroup$
    – lab bhattacharjee
    Dec 18 '18 at 10:08










  • $begingroup$
    In my case, your $d$ is my $x^d-1$, right? Sorry I want to understand clearly
    $endgroup$
    – Alessar
    Dec 18 '18 at 10:23








  • 1




    $begingroup$
    I've stated common divisor, ultimately this leads to the greatest common divisor
    $endgroup$
    – lab bhattacharjee
    Dec 18 '18 at 10:25














  • 1




    $begingroup$
    math.stackexchange.com/questions/7473/…
    $endgroup$
    – lab bhattacharjee
    Dec 18 '18 at 9:39










  • $begingroup$
    I can't understand this passage; if $n > m$, then $$gcd(a^n - 1, a^m - 1) =color{red}{ gcd(a^n - 1, a^n - a^{n-m}) = gcd(a^{n-m} - 1, a^m - 1)}$$
    $endgroup$
    – Alessar
    Dec 18 '18 at 10:02






  • 1




    $begingroup$
    If $d$ divide both, $d$ must divide $a^n-a^m=a^m(a^{n-m}-1)$ But $dnmid a^m$
    $endgroup$
    – lab bhattacharjee
    Dec 18 '18 at 10:08










  • $begingroup$
    In my case, your $d$ is my $x^d-1$, right? Sorry I want to understand clearly
    $endgroup$
    – Alessar
    Dec 18 '18 at 10:23








  • 1




    $begingroup$
    I've stated common divisor, ultimately this leads to the greatest common divisor
    $endgroup$
    – lab bhattacharjee
    Dec 18 '18 at 10:25








1




1




$begingroup$
math.stackexchange.com/questions/7473/…
$endgroup$
– lab bhattacharjee
Dec 18 '18 at 9:39




$begingroup$
math.stackexchange.com/questions/7473/…
$endgroup$
– lab bhattacharjee
Dec 18 '18 at 9:39












$begingroup$
I can't understand this passage; if $n > m$, then $$gcd(a^n - 1, a^m - 1) =color{red}{ gcd(a^n - 1, a^n - a^{n-m}) = gcd(a^{n-m} - 1, a^m - 1)}$$
$endgroup$
– Alessar
Dec 18 '18 at 10:02




$begingroup$
I can't understand this passage; if $n > m$, then $$gcd(a^n - 1, a^m - 1) =color{red}{ gcd(a^n - 1, a^n - a^{n-m}) = gcd(a^{n-m} - 1, a^m - 1)}$$
$endgroup$
– Alessar
Dec 18 '18 at 10:02




1




1




$begingroup$
If $d$ divide both, $d$ must divide $a^n-a^m=a^m(a^{n-m}-1)$ But $dnmid a^m$
$endgroup$
– lab bhattacharjee
Dec 18 '18 at 10:08




$begingroup$
If $d$ divide both, $d$ must divide $a^n-a^m=a^m(a^{n-m}-1)$ But $dnmid a^m$
$endgroup$
– lab bhattacharjee
Dec 18 '18 at 10:08












$begingroup$
In my case, your $d$ is my $x^d-1$, right? Sorry I want to understand clearly
$endgroup$
– Alessar
Dec 18 '18 at 10:23






$begingroup$
In my case, your $d$ is my $x^d-1$, right? Sorry I want to understand clearly
$endgroup$
– Alessar
Dec 18 '18 at 10:23






1




1




$begingroup$
I've stated common divisor, ultimately this leads to the greatest common divisor
$endgroup$
– lab bhattacharjee
Dec 18 '18 at 10:25




$begingroup$
I've stated common divisor, ultimately this leads to the greatest common divisor
$endgroup$
– lab bhattacharjee
Dec 18 '18 at 10:25










0






active

oldest

votes











Your Answer





StackExchange.ifUsing("editor", function () {
return StackExchange.using("mathjaxEditing", function () {
StackExchange.MarkdownEditor.creationCallbacks.add(function (editor, postfix) {
StackExchange.mathjaxEditing.prepareWmdForMathJax(editor, postfix, [["$", "$"], ["\\(","\\)"]]);
});
});
}, "mathjax-editing");

StackExchange.ready(function() {
var channelOptions = {
tags: "".split(" "),
id: "69"
};
initTagRenderer("".split(" "), "".split(" "), channelOptions);

StackExchange.using("externalEditor", function() {
// Have to fire editor after snippets, if snippets enabled
if (StackExchange.settings.snippets.snippetsEnabled) {
StackExchange.using("snippets", function() {
createEditor();
});
}
else {
createEditor();
}
});

function createEditor() {
StackExchange.prepareEditor({
heartbeatType: 'answer',
autoActivateHeartbeat: false,
convertImagesToLinks: true,
noModals: true,
showLowRepImageUploadWarning: true,
reputationToPostImages: 10,
bindNavPrevention: true,
postfix: "",
imageUploader: {
brandingHtml: "Powered by u003ca class="icon-imgur-white" href="https://imgur.com/"u003eu003c/au003e",
contentPolicyHtml: "User contributions licensed under u003ca href="https://creativecommons.org/licenses/by-sa/3.0/"u003ecc by-sa 3.0 with attribution requiredu003c/au003e u003ca href="https://stackoverflow.com/legal/content-policy"u003e(content policy)u003c/au003e",
allowUrls: true
},
noCode: true, onDemand: true,
discardSelector: ".discard-answer"
,immediatelyShowMarkdownHelp:true
});


}
});














draft saved

draft discarded


















StackExchange.ready(
function () {
StackExchange.openid.initPostLogin('.new-post-login', 'https%3a%2f%2fmath.stackexchange.com%2fquestions%2f3044953%2fshow-that-in-any-field-holds-xr-1-xs-1-xd-1-iff-d-gcdr-s%23new-answer', 'question_page');
}
);

Post as a guest















Required, but never shown

























0






active

oldest

votes








0






active

oldest

votes









active

oldest

votes






active

oldest

votes
















draft saved

draft discarded




















































Thanks for contributing an answer to Mathematics Stack Exchange!


  • Please be sure to answer the question. Provide details and share your research!

But avoid



  • Asking for help, clarification, or responding to other answers.

  • Making statements based on opinion; back them up with references or personal experience.


Use MathJax to format equations. MathJax reference.


To learn more, see our tips on writing great answers.




draft saved


draft discarded














StackExchange.ready(
function () {
StackExchange.openid.initPostLogin('.new-post-login', 'https%3a%2f%2fmath.stackexchange.com%2fquestions%2f3044953%2fshow-that-in-any-field-holds-xr-1-xs-1-xd-1-iff-d-gcdr-s%23new-answer', 'question_page');
}
);

Post as a guest















Required, but never shown





















































Required, but never shown














Required, but never shown












Required, but never shown







Required, but never shown

































Required, but never shown














Required, but never shown












Required, but never shown







Required, but never shown







Popular posts from this blog

Plaza Victoria

In PowerPoint, is there a keyboard shortcut for bulleted / numbered list?

How to put 3 figures in Latex with 2 figures side by side and 1 below these side by side images but in...